Putnam Product!

Algebra Level 3

n = 2 n 3 1 n 3 + 1 \displaystyle\prod _{ n=2 }^{ \infty }{ \frac { { n }^{ 3 }-1 }{ { n }^{ 3 }+1 } } The value of the infinite product above can expressed as p q \frac{p}{q} where p p and q q are co-prime positive integers. What is the value of p + q p+q ?


This problem appeared in Putnam - 1977.
This problem is from the set "Olympiads and Contests Around the World -1". You can see the rest of the problems here .


The answer is 5.

This section requires Javascript.
You are seeing this because something didn't load right. We suggest you, (a) try refreshing the page, (b) enabling javascript if it is disabled on your browser and, finally, (c) loading the non-javascript version of this page . We're sorry about the hassle.

3 solutions

Danny He
May 28, 2014

n 3 1 = ( n 1 ) ( n 2 + n + 1 ) n^3 - 1 = \left(n-1\right) \left(n^2+n+1\right)

n 3 + 1 = ( n + 1 ) ( n 2 n + 1 ) n^3 + 1 = \left(n+1\right) \left(n^2-n+1\right)

From this we see that n = 2 n 3 1 n 3 + 1 = n = 2 n 1 n + 1 n = 2 n 2 + n + 1 n 2 n + 1 \prod_{n=2}^\infty \frac{n^3 -1}{n^3 +1}= \prod_{n=2}^\infty \frac{n-1}{n+1}\prod_{n=2}^\infty \frac{n^2+n+1}{n^2-n+1}

Let: f ( n ) = n 1 , g ( n ) = n + 1 , h ( n ) = n 2 + n + 1 , i ( n ) = n 2 n + 1 f \left(n \right) = n-1, \: g \left(n\right) = n+1, \: h \left(n\right) = n^2 + n + 1, \: i\left(n\right) = n^2 -n +1

Clearly f ( n + 2 ) = g ( n ) f\left(n+2\right) = g\left(n\right) therefore n = 2 n 1 n + 1 = 1 3 2 4 3 5 4 6 . . . = 2 \prod_{n=2}^\infty \frac{n-1}{n+1} = \frac{1}{3}*\frac{2}{4}*\frac{3}{5}*\frac{4}{6}...=2

We also see that

i ( n + 1 ) = ( n + 1 ) 2 ( n + 1 ) + 1 i\left(n+1\right) = \left(n+1\right)^2 - \left(n+1\right) + 1

= n 2 + 2 n + 1 n 1 + 1 = n^2 + 2n + 1 -n - 1 + 1

= n 2 + n + 1 = n^2 + n +1

= h ( n ) = h\left(n\right)

Therefore n = 2 n 2 + n + 1 n 2 n + 1 = 7 3 13 7 21 13 . . . = 1 3 \prod_{n=2}^\infty \frac{n^2+n+1}{n^2-n+1} = \frac{7}{3}*\frac{13}{7}*\frac{21}{13}... = \frac{1}{3}

Hence we get n = 2 n 3 1 n 3 + 1 = 2 3 \prod_{n=2}^\infty \frac{n^3-1}{n^3+1} = \frac{2}{3}

So we get p + q = 2 + 3 = 5 p+q = 2+3 = 5

Jubayer Nirjhor
May 3, 2014

n = 2 n 3 1 n 3 + 1 = lim N [ n = 2 N n 3 1 n 3 + 1 ] = lim N [ n = 2 N ( n 1 ) ( n 2 + n + 1 ) ( n + 1 ) ( n 2 n + 1 ) ] = lim N [ n = 2 N n 1 n + 1 n = 2 N n 2 + n + 1 n 2 n + 1 ] = lim N [ 2 ( N 1 ) ! ( N + 1 ) ! n = 2 N n 2 + n + 1 n 2 n + 1 ] = lim N [ 2 N ( N + 1 ) n = 2 N n 2 + n + 1 n 2 n + 1 ] \begin{aligned} \prod_{n=2}^\infty \dfrac{n^3-1}{n^3+1} &=&\lim_{N\to \infty} \left[\prod_{n=2}^N \dfrac{n^3-1}{n^3+1}\right] \\ &=&\lim_{N\to \infty} \left[\prod_{n=2}^N \dfrac{(n-1)(n^2+n+1)}{(n+1)(n^2-n+1)}\right] \\ &=& \lim_{N\to \infty} \left[\prod_{n=2}^N \dfrac{n-1}{n+1}\prod_{n=2}^N \dfrac{n^2+n+1}{n^2-n+1}\right] \\ &=& \lim_{N\to \infty} \left[\dfrac{2(N-1)!}{(N+1)!}\prod_{n=2}^N \dfrac{n^2+n+1}{n^2-n+1}\right] \\ &=& \lim_{N\to \infty} \left[\dfrac{2}{N(N+1)}\prod_{n=2}^N \dfrac{n^2+n+1}{n^2-n+1}\right] \end{aligned}

Since: n 2 + n + 1 = ( n + 1 ) 2 ( n + 1 ) + 1 n^2+n+1=(n+1)^2-(n+1)+1 , the latter product telescopes too (all the terms except the first index term and the last index term).

lim N [ 2 N ( N + 1 ) n = 2 N n 2 + 1 + n n 2 + 1 n ] = lim N [ 2 N ( N + 1 ) × N 2 + N + 1 3 ] = lim N [ 2 3 × N 2 + N + 1 N 2 + N ] = lim N [ 2 3 ( 1 + 1 N 2 + N ) ] = 2 3 p q \begin{aligned} \lim_{N\to \infty} \left[\dfrac{2}{N(N+1)}\prod_{n=2}^N \dfrac{n^2+1+n}{n^2+1-n}\right] &=& \lim_{N\to \infty} \left[\dfrac{2}{N(N+1)}\times\dfrac{N^2+N+1}{3}\right] \\ &=& \lim_{N\to\infty} \left[\dfrac{2}{3} \times \dfrac{N^2+N+1}{N^2+N}\right] \\ &=& \lim_{N\to\infty} \left[\dfrac{2}{3} \left(1+\dfrac{1}{N^2+N}\right)\right] \\ &=& \boxed{\dfrac{2}{3}} \equiv \dfrac{p}{q}\\ \end{aligned}

p + q = 2 + 3 = 5 \therefore ~~~ p+q=2+3=\fbox{5}

I saw this problem in Problem Solving Strategies by Arthur Engel. I had already seen it and so knew the answer beforehand. :P

Anik Chakrabarty - 7 years ago
Kartik Sharma
May 27, 2014

n = 2 n 3 1 n 3 + 1 \prod _{ n=2 }^{ \infty }{ \frac { { n }^{ 3 }-\quad 1 }{ { n }^{ 3 }\quad +\quad 1 } }

= 2 3 1 2 3 + 1 × 3 3 1 3 3 + 1 × . . . . . . . . \frac { { 2 }^{ 3 }\quad -\quad 1 }{ { 2 }^{ 3 }\quad +\quad 1 } \times \frac { { 3 }^{ 3 }\quad -\quad 1 }{ { 3 }^{ 3 }\quad +\quad 1 } \times ........\infty [ \prod _{ }^{ }{ } means product of each term]

Using formula a^3 - b^3 = (a-b)(a^2 + b^2 +ab) and a^3 + b^3 = (a+b)(a^2 + b^2 -ab),

= ( 2 1 ) ( 2 2 + 1 2 + 2 1 ) ( 2 + 1 ) ( 2 2 + 1 2 2 1 ) × ( 3 1 ) ( 3 2 + 1 2 + 3 1 ) ( 3 + 1 ) ( 3 2 + 1 2 3 1 ) × ( 4 1 ) ( 4 2 + 1 2 + 4 1 ) ( 4 + 1 ) ( 4 2 + 1 2 4 1 ) × . . . . \frac { (2-1)({ 2 }^{ 2 }\quad +\quad { 1 }^{ 2 }\quad +2*1) }{ (2+1)({ 2 }^{ 2 }\quad +\quad { 1 }^{ 2 }\quad -2*1) } \times \frac { (3-1)({ 3 }^{ 2 }\quad +\quad { 1 }^{ 2 }\quad +3*1) }{ (3+1)({ 3 }^{ 2 }\quad +\quad { 1 }^{ 2 }\quad -3*1) } \times \frac { (4-1)({ 4 }^{ 2 }\quad +\quad { 1 }^{ 2 }\quad +4*1) }{ (4+1)({ 4 }^{ 2 }\quad +\quad { 1 }^{ 2 }\quad -4*1) } \times ....\infty

On simplifying, we get,

= 1 × 7 3 × 3 × 2 × 13 4 × 7 × 3 × 21 5 × 13 × . . . . \frac { 1\times 7 }{ 3\times 3 } \times \frac { 2\times 13 }{ 4\times 7 } \times \frac { 3\times 21 }{ 5\times 13 } \times ....\infty

therefore, 7 and 7, 3 and 3, 13 and 13 will cut and this trend will go on forever as it is an infinite series and so, every term will cut except 2 and 3

= 2 3 \boxed { \frac { 2 }{ 3 } }

Hence, the answer is 5

Did it exactly in the same way. Cheers!. Did U use latex for this?

Krishna Ar - 7 years ago

Hey ...why don't you follow me and then I'll follow you back?

Krishna Ar - 7 years ago

0 pending reports

×

Problem Loading...

Note Loading...

Set Loading...